Suppose $a_1, a_2, \ldots, a_n$ and $b_1, b_2, \ldots, b_n$ are real numbers such that \[ (a_1 ^ 2 + a_2 ^ 2 + \cdots + a_n ^ 2 -1)(b_1 ^ 2 + b_2 ^ 2 + \cdots + b_n ^ 2 - 1) > (a_1 b_1 + a_2 b_2 + \cdots + a_n b_n - 1)^2. \] Prove that $a_1 ^ 2 + a_2 ^ 2 + \cdots + a_n ^ 2 > 1$ and $b_1 ^ 2 + b_2 ^ 2 + \cdots + b_n ^ 2 > 1$.
Problem
Source: USA TST 2004
Tags: inequalities, quadratics, algebra, polynomial, trigonometry, geometry, parallelogram
18.03.2006 08:09
20.03.2006 13:43
fuzzylogic wrote:
eh, if conclusion is assumed false, then either the first or the second of what you said is true, not necessarily both.
20.03.2006 15:20
al.M.V. wrote: fuzzylogic wrote:
eh, if conclusion is assumed false, then either the first or the second of what you said is true, not necessarily both. If only one is true, then the L.H.S would be negative, whereas the R.H.S is positive, hence either neither or both have to be true.
16.12.2007 06:44
Another solution: Suppose instead that $ a_{1}^2 + ... + a_{n}^2 \leq 1$ and $ b_{1}^2 + ... + b_{n}^2 \leq 1$. We shall prove that $ (a_{1}^2 + ... + a_{n}^2- 1)(b_{1}^2 + ... + b_{n}^2 - 1) \leq (a_{1}b_{1} + ... + a_{n}b_{n} - 1)^2$. Noting that Cauchy-Schwarz is valid for any, (not necessarily positive) reals, we have: $ (a_{1}b_{1} + ... + a_{n}b_{n})^2 \leq (a_{1}^2 + ... + a_{n}^2)(b_{1}^2 + ... + b_{n}^2) \leq 1$, so $ a_{1}b_{1} + ... + a_{n}b_{n} \leq |a_{1}b_{1} + ... + a_{n}b_{n}| \leq 1$. Let $ X = a_{1}^2 + ... + a_{n}^2, Y = b_{1}^2 + ... + b_{n}^2$, so $ 0 \leq X,Y \leq 1$. Since $ 0 \leq X+Y - 2\sqrt {XY}$, we have $ XY - (X+Y) \leq XY - 2 \sqrt {XY}$, so $ (X-1)(Y-1) \leq (\sqrt{XY} - 1)^2$, (1) But Cauchy-Schwarz gives us $ a_{1}b_{1} + ... + a_{n}b_{n} \leq \sqrt {XY}$, so since $ f(x) = (x-1)^2$ is decreasing for $ x \leq 1$, $ ( \sqrt {XY} - 1)^2 \leq (a_{1}b_{1} + ... + a_{n}b_{n} - 1)^2$ (2). Combining (1) and (2) gives us our desired inequality.
24.02.2012 23:56
Let $A = a_1^2 + a_2^2 + \cdots + a_n^2$, $B = b_1^2 + b_2^2 + \cdots + b_n^2$, and $C = a_1b_1 + a_2 b_2 + \cdots + a_n b_n$. We are given that $(A-1)(B-1) > (C-1)^2$, which can be rearranged to $AB - C^2 > A+B - 2C$. Rewrite this inequality as $(\sqrt{AB} - C)(\sqrt{AB}+C) > (\sqrt{A} - \sqrt{B})^2 + 2(\sqrt{AB} - C)$, so $(\sqrt{AB}-C)(\sqrt{AB}+C-2) > (\sqrt{A} - \sqrt{B})^2 \geq 0$. By the Cauchy-Schwarz inequality, $\sqrt{AB} \geq C$ so we need $\sqrt{AB}+C-2 \geq 0$. But $C \leq \sqrt{AB}$, so $2 \sqrt{AB} \geq 2 \implies AB \geq 1$. Since $(A-1)(B-1) > 0$, either both $A<1,B<1$ or both $A>1,B>1$. In the former case, we have $AB < 1$, a contradiction, so we must have $A,B>1$, as desired.
26.02.2012 15:07
I'm sorry, but what is Aczel's Inequality?
27.02.2012 22:59
Aczel's Inequality states that if $A^2>a_1^2+a_2^2+\cdots+a_n^2$ and $B_2>b_1^2+b_2^2+\cdots+b_n^2$, then $\left(A^2-a_1^2-a_2^2-\cdots-a_n^2\right)\left(B^2-b_1^2-b_2^2-\cdots-b_n^2\right)\le$ $\left(AB-a_1b_1-a_2b_2-\cdots-a_nb_n\right)$. See the page about it on AoPSWiki.
25.11.2014 23:07
02.05.2019 09:15
Let $\mathbf{a}=\begin{bmatrix} a_1 \\ \vdots \\ a_n \end{bmatrix}$ and $\mathbf{b}=\begin{bmatrix} b_1 \\ \vdots \\ b_n \end{bmatrix}$. Then \begin{align*} ||\mathbf{a}-\mathbf{b}||^2||\mathbf{a}||^2&=||\mathbf{a}||^2||\mathbf{b}||^2+||\mathbf{a}-\mathbf{b}||^2||\mathbf{a}||^2-||\mathbf{a}||^2||\mathbf{b}||^2\\ &=||\mathbf{a}||^2||\mathbf{b}||^2+\left(||\mathbf{a}-\mathbf{b}||^2-||\mathbf{b}||^2\right)||\mathbf{a}||^2\\ &=||\mathbf{a}||^2||\mathbf{b}||^2+\left\langle\mathbf{a}-2\mathbf{b},\mathbf{a}\right\rangle\cdot\left\langle\mathbf{a},\mathbf{a}\right\rangle\\ &=||\mathbf{a}||^2||\mathbf{b}||^2+\left\langle\mathbf{a}-\mathbf{b},\mathbf{a}\right\rangle^2-\left\langle\mathbf{b},\mathbf{a}\right\rangle^2\\ &\geq||\mathbf{a}||^2||\mathbf{b}||^2-\left\langle\mathbf{a},\mathbf{b}\right\rangle^2\\ &=\left(||\mathbf{a}||^2-1\right)\left(||\mathbf{b}||^2-1\right)-\left\langle\mathbf{a},\mathbf{b}\right\rangle^2+||\mathbf{a}||^2+||\mathbf{b}||^2-1\\ &>\left(\left\langle\mathbf{a},\mathbf{b}\right\rangle-1\right)^2-\left\langle\mathbf{a},\mathbf{b}\right\rangle^2+||\mathbf{a}||^2+||\mathbf{b}||^2-1\\ &=||\mathbf{a}||^2-2\left\langle\mathbf{a},\mathbf{b}\right\rangle+||\mathbf{b}||^2\\ &=||\mathbf{a}-\mathbf{b}||^2 \end{align*}so $||\mathbf{a}||>1$. Similarly $||\mathbf{b}||>1$. Thus $a_1^2+\ldots+a_n^2>1$ and $b_1^2+\ldots+b_n^2>1$.
12.12.2020 12:59
http://www.artofproblemsolving.com/Wiki/index.php/Aczel's_Inequality. kill by Aczel inequality
14.04.2021 21:31
Let $a=\sum_{i=1}^{n} a_i^2 , b=\sum_{i=1}^{n} b_i^2 , c=\sum_{i=1}^{n} a_ib_i$. Assume for contradiction that $a<1$ and $b<1$. Given inequality can be written as $c^2 - 2c + (a + b - ab) < 0$. The function $f(x)=x^2 - 2x + (a + b - ab)$ has $2$ positive roots one of which is less than $1$ and that root is $1-\sqrt{(a-1)(b-1)}$. So $c > 1 - \sqrt{(a-1)(b-1)}$. By Cauchy we know that $ab\geq c^2 > (a-1)(b-1) + 1 - 2\sqrt{(a-1)(b-1)} \implies a + b + 2\sqrt{(a-1(b-1)} > 2$. But it can easily be proven by computing derivatives that when $0<a<1$ and $0<b<1$, $a + b + 2\sqrt{(a-1)(b-1)}$ takes its maximum value $2$ when $a=b$. Hence contradiction.
16.04.2021 23:07
Apparently, essentially the same as an early problem from 2000 Mongolian Olympiad, Teacher-secondary level. See here.
17.04.2021 16:51
there's a nice proof of aczel's inequality by constructing yet another quadratic function
04.11.2024 16:04
Aczel's Inequality with $A=B=1$.